~(p->q) of part of A Probabilistic Proof of Wallis's Formula for pi

  • Thread starter Thread starter Dustinsfl
  • Start date Start date
  • Tags Tags
    Formula Pi Proof
Click For Summary
The discussion focuses on the negation of a theorem regarding nonnegative continuous functions and their integrals. The original statement asserts that if g(x) is a nonnegative continuous function with a finite integral, then a scaled version can serve as a probability distribution. The negation of the implication P implies Q is clarified as not P or Q, leading to confusion about how to properly negate P. It is concluded that the negation of P should encompass all conditions, resulting in the statement that g(x) is either not continuous, takes on negative values, or has a divergent integral. The clarification emphasizes the logical structure of the negation in the context of the theorem.
Dustinsfl
Messages
2,217
Reaction score
5

Homework Statement


Write in symbols the negation of the theorem stated in part a.

part a:We immediately see that if g(x) is a nonnegative continuous function whose integral is finite, then there exists an a>0 such that a*g(x) is a continuous probability distribution (take a=1/\int g(x)dx from -\infty to \infty).

The negation of P implies Q is equivalent to not P or Q.

The issue is with the P part.

Is ~P= g(x) isn't a nonnegative continuous fuction whose integral isn't finite?

I am not sure if both is parts need to be negated or just one.

Then for the Q part it is just itself starting with the existential quantifier.
 
Physics news on Phys.org
Dustinsfl said:
The negation of P implies Q is equivalent to not P or Q.

~(P => Q) is equivalent to "P and not Q."

Since you don't need the negation of P, does that solve your problem?
 
Last edited:
Just for completeness sake:

I suppose P is the statement "g(x) is a non-negative continuous function whose integral is finite." I would re-write this as "g(x) is a function which is non-negative and continuous and whose integral is finite" to emphasize the internal logical structure. Since ~(a and b and c) = ~a or ~b or ~c, the negation of P is:

"g(x) is a function which is not continuous or assumes negative values or whose integral diverges."
 
Question: A clock's minute hand has length 4 and its hour hand has length 3. What is the distance between the tips at the moment when it is increasing most rapidly?(Putnam Exam Question) Answer: Making assumption that both the hands moves at constant angular velocities, the answer is ## \sqrt{7} .## But don't you think this assumption is somewhat doubtful and wrong?

Similar threads

Replies
2
Views
2K
  • · Replies 3 ·
Replies
3
Views
502
  • · Replies 1 ·
Replies
1
Views
3K
  • · Replies 7 ·
Replies
7
Views
2K
  • · Replies 6 ·
Replies
6
Views
2K
  • · Replies 4 ·
Replies
4
Views
1K
  • · Replies 12 ·
Replies
12
Views
3K
Replies
9
Views
6K
  • · Replies 10 ·
Replies
10
Views
2K
  • · Replies 100 ·
4
Replies
100
Views
12K